Number of invertible symmetric $3\times 3$ matrices over a finite field $F =\mathbb{F}_q$

abstract-algebrafinite-fieldsfinite-groupsmatrices

I was trying to find the number of symmetric invertible matrices of size $n\times n$ over a finite field $F= \mathbb{F}_q$ ( $q$ odd for simplicity)

For $n=2$ it has to do with counting the number of zeroes of a quadratic form over a finite field, something that was seen before on this site.

For $n = 3$ it is a completely new problem.

I am trying it in this way:

The first subset of (symmetric) invertible matrices are the one having a Gauss decomposition

$$A = L D U$$

where $L$ is lower triangular unipotent, $U$ is upper triangular unipotent, and $D$ is diagonal with non-zero diagonal values. If $A$ is moreover symmetric then $L = U^t$. Now the number of such matrices is simple to determine. The matrices with a Gauss decomposition are those for which $D_1$, $\ldots$, $D_n \ne 0$. where $D_k$ are the leading minors, $1\le k \le n$. So we got a subset of the set of symmetric invertible matrices.

Let's try to determine the number of $n\times n$ symmetric invertible matrices. Now, let $d_{n-1}$ the number of symmetric $(n-1)\times (n-1)$ matrices that are invertible. From this we can complete the off diagonal elements in $q^{n-1}$ ways, and the last diagonal element in $(q-1)$ ways ( use the Cauchy formula for expanding the determinant).

The problem is what happens if the $D_{n-1}$ leading minor is in fact zero. Now its rank starts to matter. Perhaps I could cover the case $n=3$, but it looks a bit confusing.

Notes: I am aware there exists a paper that solves this problem ( symmetric matrices with $0$ determinant over the ring $\mathbb{Z}_m$), but it seems hard to read. Even inequalities would be helpful.

Any feedback would be welcome!

Best Answer

The linked paper only seems to address the question directly for the case that $q$ is itself prime (in the notation of that reference, $\mu = 1$), since $\Bbb Z_{p^a} \not\cong \Bbb F_{p^a}$ for $p$ prime and $a > 1$.

In any case, the linked paper's formula simplifies substantially in the case that $q$ is prime (which we'll assume herein, as well as that $q > 2$), and in fact that case was resolved a few decades earlier in:

Carlitz, L. Representations by quadratic forms in a finite field. Duke Math. J. (1954) 21(1), 123–137. doi:10.1215/s0012-7094-54-02114-6

In particular, the coefficients $T_\beta(k, s) = T_\beta(k, 0)$ in the linked paper of Brent & McKay are identically $1$, as is the leading coefficient of equation (5.2) there. That equation gives the proportion $P(n, q)$ of symmetric $n \times n$ matrices over $\Bbb F_q$ that are nondegenerate, and there are $q^{{n + 1} \choose 2}$ symmetric matrices, so multiplying gives a general formula for the count $q^{{n + 1} \choose 2} P(n, q)$ that depends on the parity of $n$. (Unfortunately, the linked paper instead denotes the prime by $p$ and defines $q = \frac{1}{p}$. The below formulae are instead consistent with the question statement.)

  • If $n$ is even there are $$q^{{n + 1} \choose 2} \frac{\prod_{j = 1}^n (1 - q^{-j})}{\prod_{j = 1}^{n / 2} (1 - q^{-2 j})}$$ invertible, symmetric matrices over $\Bbb F_q$.
  • If $n$ is odd there are $$q^{{n + 1} \choose 2} (1 - q^{-n}) \frac{\prod_{j = 1}^{n - 1} (1 - q^{-j})}{\prod_{j = 1}^{(n - 1) / 2} (1 - q^{-2 j})} $$ invertible, symmetric matrices over $\Bbb F_q$.

In the special case that $n = 3$, there are $$(q^3 - 1) (q^3 - q^2) = q^6 - q^5 - q^3 + q^2$$ invertible, symmetric matrices over $\Bbb F_q$. A quick Maple script verifies this formula for $q = 3, 5, 7$. (This count is $\frac{1}{q^3 - q}$ times the number of invertible $3 \times 3$ matrices---not necessarily symmetric---over $\Bbb F_q$, and some meditation on that fact might suggest a slick way to produce the above formula.)